LSAT and Law School Admissions Forum

Get expert LSAT preparation and law school admissions advice from PowerScore Test Preparation.

 Administrator
PowerScore Staff
  • PowerScore Staff
  • Posts: 8929
  • Joined: Feb 02, 2011
|
#34544
Please post your questions below! Thanks!
 gen2871
  • Posts: 47
  • Joined: Jul 01, 2018
|
#48168
Hi, the reason C is correct is because the answer choice yields the following diagram: "test drive people who are decided to buy dont buy :arrow: some fault become evident. The necessary condition directly attacks the "quality of the car is not that impressive as the question explicitly states?

If the question stem asks which of the following weakens the argument, I wonder if D is the correct answer. The fact 90% dont end up buying the car contradicts of over 80% ppl test drives buy. Please advise. Thank you.
 Adam Tyson
PowerScore Staff
  • PowerScore Staff
  • Posts: 5191
  • Joined: Apr 14, 2011
|
#48181
I wouldn't recommend tackling this question with a conditional analysis, gen2871, because it is not clearly conditional, and in fact seems to be more of a causal argument as indicated by the stem: "If the advertisement is interpreted as implying that the quality of the car is unusually impressive". This is saying that if someone were to believe that the test drive is actually causing people to buy the car, we want the answer that would suggest that it is not necessarily the cause of the purchase decision.

Answer D doesn't hurt the argument because the author never said, or even suggested, that those who test drive the car will buy one the same day. It also doesn't matter because it isn't about Zenith cars but about ALL cars. So maybe 90% of all car buyers delay the purchase decision, but Zenith customers are so impressed that they buy it on the spot? The ones who are slower to decide are test driving other cars.

Answer C weakens the argument by suggesting that the causal relationship is reversed - they don't buy because of the test drive, but test drive because they intend to buy! A reversed cause and effect will always undermine a causal claim!

Avoid going into conditional analyses unless the argument is clearly conditional, without much effort to paraphrase is in those "if...then" terms. When causal indicators are present, or even when a causal relationship is strongly implied, use that form of analysis instead.

Good luck!
 gen2871
  • Posts: 47
  • Joined: Jul 01, 2018
|
#48196
Ah! Thank you so very much for filling me with your wisdom, Adam! On the side note, I have been diagraming since.... then.... as conditional forever. You just resolved my other mischief. You are truly awesome and impressive!
 whardy21
  • Posts: 48
  • Joined: Sep 30, 2018
|
#65746
I don't see the casual reasoning in this question. Please explain.
User avatar
 Dave Killoran
PowerScore Staff
  • PowerScore Staff
  • Posts: 5862
  • Joined: Mar 25, 2011
|
#65774
Check out the question stem for the causal angle, as noted by Adam above: "If the advertisement is interpreted as implying that the quality of the car is unusually impressive..." They are saying that the advertisement should be interpreted as claiming the quality of the car causes people to buy it.

Thanks!
 lyn
  • Posts: 11
  • Joined: Jan 27, 2020
|
#73577
I understand why C is correct and that was the answer I was initially going to choose. However I don't understand why E is incorrect.

The question stem specifically asks for which answer casts doubt on the fact that the Zenith Car is of unusually impressive quality. Shouldn't the fact that a part is broken within the first year show that the car is in fact not of an impressive quality? If that is the question that the stem is asking, then i'm thinking answer E will be more correct than answer C.

Please Clarify,

Thanks
User avatar
 KelseyWoods
PowerScore Staff
  • PowerScore Staff
  • Posts: 1079
  • Joined: Jun 26, 2013
|
#73615
Hi Lyn!

The advertisement says that over 80 percent of people who test-drive a Zenith end up buying one, suggesting that the quality of the car as perceived during the test-drive is what is unusually impressive enough to cause people to buy the car. Answer choice (E) talks about a minor part breaking within the first year...but this is not something that people would notice during the test-drive and it's just a minor part, which may not greatly impact their perception of the overall quality of the car anyway. We're not just trying to cast doubt on the quality of the car, we're trying to cast doubt on the implication that the quality of the car during the test-drive is so impressive that people buy it for that reason. Answer choice (E) doesn't cast doubt on the idea that the quality of the car during the test-drive is impressive and doesn't give an alternate reason for why people people might purchase the car after test-driving besides its impressive qualities.

Hope this helps!

Best,
Kelsey
 lyn
  • Posts: 11
  • Joined: Jan 27, 2020
|
#73627
Hi Kelsey,

Thanks for the clarification. I guess my mistake was I was enlarging the scope of the question. If you limit it to just the test drive, then yes answer E would obviously be incorrect.

Thank you so much!

Get the most out of your LSAT Prep Plus subscription.

Analyze and track your performance with our Testing and Analytics Package.